UTNianos

Versión completa: Intervalo de CV
Actualmente estas viendo una versión simplificada de nuestro contenido. Ver la versión completa con el formato correcto.
(31-10-2013 23:19)Bian escribió: [ -> ]Ayer estaba haciendo un ej, y habia un limite que me quedaba lim de n->infinito [(n+1)*ln(n+1)]/[n*ln(n)] y yo probe por ej juntando los logaritmos, y sacando el ln afuera del limite asi:

lim n->inf (n+1)/(n) * ln [ lim n->inf (n+1/n) ] , y no me queda igual que si lo resuelvo por L'Hopital (que no me da 0) en cambio de esta forma da 0, y está mal.. tenes idea de por qué no lo puedo separar así? porque me quede re pensando en ese

lo mas conveniente es que por cada ejercicio inicies un nuevo th asi no hacemos uno extenso dale ???

por lo poco que puedo entender (si podes usar latex te lo agradezco, o sino una imagen alcanza thumbup3 ) vos hiciste

\[\frac{ln(a+b)}{ln(a)}=\ln\left(\frac{a+b}{a}\right)\]

esa propiedad no existe , de donde la sacaste ??.. esta bien si separas como un producto

\[\lim_{n\to\infty}\left(\frac{n+1}{n} \cdot \frac{\ln(n+1)}{ln(n)} \right )\]

el limite de un producto es el producto de los limites, si ambos existen entonces el limite del producto existe , entonces

\[\lim_{n\to\infty}\left(\frac{n+1}{n}\left)\cdot \lim_{n\to\infty}\left(\frac{\ln(n+1)}{ln(n)} \right )\]

el primero tiende a 1 ahora para el segundo, si al argumento del logaritmo en el numerador multiplico y divido por n

\[\left(\frac{\ln(n+1)}{ln(n)}\right)=\left(\frac{\ln\left((n+1)\cdot\dfrac{n}{n}\right)}{ln(n)}\right)\]

aplico propiedad del producto de logaritmos

\[\left(\frac{\ln\left((n+1)\cdot\dfrac{n}{n}\right)}{ln(n)}\right)=\frac{\ln\left(\dfrac{n+1}{n}\right)+\ln n}{\ln n}\]

si distribuyo el denominador

\[\frac{\ln\left(\dfrac{n+1}{n}\right)+\ln n}{\ln n}=\frac{\ln\left(\dfrac{n+1}{n}\right)}{\ln n}+1\]

aplico limites, y por propiedad de la suma

\[\lim_{n\to \infty}\frac{\ln\left(\dfrac{n+1}{n}\right)}{\ln n}+\lim_{n\to\infty}1\]

luego, lo que esta en el argumento es siempre positivo, la base es mayor a cero, entonces

\[\lim_{n\to \infty}\underbrace{\frac{\overbrace{\ln\left(\underbrace{\dfrac{n+1}{n}\right}_{1}\right)}^{0}}{\underbrace{\ln n}_{\infty}}}_0+1=1\]

finalmente

\[\lim_{n\to\infty}\left(\frac{n+1}{n} \cdot \frac{\ln(n+1)}{ln(n)} \right )=1\cdot 1=1\neq 0\]

por la tanto la serie no CV
ahhhh con razon a mi no me daba cuando separaba los limites, pero si cuando lo dejaba asi, porque estaba juntando el logaritmo! estaba re segura que ln a / ln b = ln(a/b), la verdad que no se de donde lo saque =P gracias!!
URLs de referencia